0
$\begingroup$

Related to FLT and this question.

For natural $n > 4 $ define the curve $C_n : z^{n-2}y(y+z)=x^n$.

$C_n$ has the trivial points with $x=0$ for all $n$.

The answer in the linked question shows there are no other solutions using FLT.

I am interested in elementary approach without using FLT or modularity.

Q1 Do we get constraints or lack of non-trivial coprime integral points $(X,Y,Z)$ on $C_n$ via elementary approach not using modularity?

Partial results, could be wrong. Assume $(X,Y,Z)$ is coprime point.

If prime $p$ divides $Z$ or $Y$ then it divides $X$ too.

Assume $p \mid Y$ and $p^2 \nmid Y$. Since $p \mid X$ then $p$ is coprime to $Z$. Set $Y=p Y_1, X=p X_1$. Then $Z^{n-2} p Y_1 (p Y_1 + Z)=(p X_1)^n=p^n X_1^n$. The LHS is divisible by $p$ and is not divisible by $p^2$ and the RHS is divisible by $p^n$ and we can't equate the exponents of $p$.

Likewise, assume $p \mid Z$ and $p^2 \nmid Z$. Since $p \mid X$ then $p$ is coprime to $Y$. Set $Z=p Z_1, X=p X_1$. Then $p^{n-2} Z_1^{n-2} Y (Y + p Z_1)=(p X_1)^n=p^n X_1^n$. The LHS is divisible by $p^{n-2}$ and is not divisible by $p^{n-1}$ and the RHS is divisible by $p^n$ and we can't equate the exponents of $p$.

Q2 Is this correct proof that the non-trivial points are with $Y,Z$ powerful integers?

$\endgroup$
8
  • $\begingroup$ If $(X,Y,Z)$ was a nontrivial solution, then $x=X/Z,y=Y/Z$ would satisfy $x^n=y(y+1)$. How does the answer to the other question not answer this one? $\endgroup$
    – Wojowu
    May 24, 2019 at 14:35
  • $\begingroup$ @Wojowu Indeed. I am mainly interested in extending the partial results, if this gets closed will move it in the other question. $\endgroup$
    – joro
    May 24, 2019 at 14:39
  • $\begingroup$ What partial results? There are no solutions, what else is there to say? $\endgroup$
    – Wojowu
    May 24, 2019 at 14:40
  • $\begingroup$ @Wojowu Thanks, will edit! The other answer assumes FLT and I am interested in elementary proof, not using heavy stuff like FLT and modularity. $\endgroup$
    – joro
    May 24, 2019 at 15:20
  • $\begingroup$ @Wojowu I edited, does it make sense now? $\endgroup$
    – joro
    May 24, 2019 at 15:28

0

Your Answer

By clicking “Post Your Answer”, you agree to our terms of service and acknowledge you have read our privacy policy.